Difference between revisions of "2021 IMO Problems/Problem 6"

m
Line 1: Line 1:
==Problem==
+
== Problem ==
 
Let <math>m\ge 2</math> be an integer, <math>A</math> be a finite set of (not necessarily positive) integers, and <math>B_1,B_2,B_3,...,B_m</math> be subsets of <math>A</math>. Assume that for each <math>k = 1, 2,...,m</math> the sum of the elements of <math>B_k</math> is <math>m^k</math>. Prove that <math>A</math> contains at least <math>m/2</math> elements.
 
Let <math>m\ge 2</math> be an integer, <math>A</math> be a finite set of (not necessarily positive) integers, and <math>B_1,B_2,B_3,...,B_m</math> be subsets of <math>A</math>. Assume that for each <math>k = 1, 2,...,m</math> the sum of the elements of <math>B_k</math> is <math>m^k</math>. Prove that <math>A</math> contains at least <math>m/2</math> elements.
  
==Video solution==
+
== Video solution ==
 
https://youtu.be/vUftJHRaNx8 [Video contains solutions to all day 2 problems]
 
https://youtu.be/vUftJHRaNx8 [Video contains solutions to all day 2 problems]
 +
 +
== See also ==
 +
 +
[[Category:Olympiad Combinatorics Problems]]

Revision as of 10:35, 18 June 2023

Problem

Let $m\ge 2$ be an integer, $A$ be a finite set of (not necessarily positive) integers, and $B_1,B_2,B_3,...,B_m$ be subsets of $A$. Assume that for each $k = 1, 2,...,m$ the sum of the elements of $B_k$ is $m^k$. Prove that $A$ contains at least $m/2$ elements.

Video solution

https://youtu.be/vUftJHRaNx8 [Video contains solutions to all day 2 problems]

See also